Which are the pivot columns? - Linear Algebra

Click For Summary
The discussion revolves around identifying pivot columns in a given matrix after performing elimination. The user initially believes there are three pivots (1, -3, -6) based on their elimination process but is informed that only two pivots (1 and -3) exist, corresponding to columns 1 and 2. The confusion arises from the interpretation of pivot definitions, where a pivot must be the first non-zero entry in a row with no non-zero entries below it. A suggestion is made that the user may have incorrectly copied the matrix, which could clarify the discrepancy in the number of pivots. Understanding the correct identification of pivot columns is crucial for solving linear systems accurately.
tigrus
Messages
1
Reaction score
0

Homework Statement


Given matrix

A=

1 1 -2 1 3
2 -1 2 2 6
3 2 -4 -3 -9

x=

x1
x2
x3
x4
x5

b =

1
2
3

1. Solve the system by elimination (use the augmented matrix) until pivots are found (no backward elimination here).

2. Show that pivots are respectively 1 and -3, and indicate clearly which are the pivot columns and free columns.


Homework Equations



Matrix A and vector b



The Attempt at a Solution



I perform elimination on augmented matrix to solve part 1.

1 1 -2 1 3 | 1
0 -3 6 0 0 | 0
0 -1 2 -6 -18 | 0


1 1 -2 1 3 | 1
0 -3 6 0 0 | 0
0 0 0 -6 -18 | 0

What I don't understand is part 2.

As far as I know, for an entry to become a pivot it must be the first non-zero entry on the row and there must be no non-zero entries under it.

1 1 -2 1 3 | 1
0 -3 6 0 0 | 0
0 0 0 -6 -18 | 0

If that rule applies I would find three pivots instead of two. (1,-3, -6) I know the right answer for this exercise is that there are only two pivots (1, -3), resulting col1 & col2 to be the pivot columns and the rest are free columns.

Can someone explain why -6 is not a pivot in this case?
 
Physics news on Phys.org
tigrus said:

Homework Statement


Given matrix

A=

1 1 -2 1 3
2 -1 2 2 6
3 2 -4 -3 -9

x=

x1
x2
x3
x4
x5

b =

1
2
3

1. Solve the system by elimination (use the augmented matrix) until pivots are found (no backward elimination here).

2. Show that pivots are respectively 1 and -3, and indicate clearly which are the pivot columns and free columns.


Homework Equations



Matrix A and vector b



The Attempt at a Solution



I perform elimination on augmented matrix to solve part 1.

1 1 -2 1 3 | 1
0 -3 6 0 0 | 0
0 -1 2 -6 -18 | 0


1 1 -2 1 3 | 1
0 -3 6 0 0 | 0
0 0 0 -6 -18 | 0

What I don't understand is part 2.

As far as I know, for an entry to become a pivot it must be the first non-zero entry on the row and there must be no non-zero entries under it.

1 1 -2 1 3 | 1
0 -3 6 0 0 | 0
0 0 0 -6 -18 | 0

If that rule applies I would find three pivots instead of two. (1,-3, -6) I know the right answer for this exercise is that there are only two pivots (1, -3), resulting col1 & col2 to be the pivot columns and the rest are free columns.

Can someone explain why -6 is not a pivot in this case?
I don't see an error on your part. Are you sure you have copied the matrix correctly?
 
as far as I could tell, there should be 3 pivots in your reduced matrix.

I suspect that you wrote the matrix down wrong, because if you make the last row
-3 3 -4 -3 -9 instead of 3 3 -4 -3 -9, then there should be only two pivots.
 
Question: A clock's minute hand has length 4 and its hour hand has length 3. What is the distance between the tips at the moment when it is increasing most rapidly?(Putnam Exam Question) Answer: Making assumption that both the hands moves at constant angular velocities, the answer is ## \sqrt{7} .## But don't you think this assumption is somewhat doubtful and wrong?

Similar threads

  • · Replies 2 ·
Replies
2
Views
2K
  • · Replies 10 ·
Replies
10
Views
3K
  • · Replies 2 ·
Replies
2
Views
2K
  • · Replies 2 ·
Replies
2
Views
2K
  • · Replies 7 ·
Replies
7
Views
1K
  • · Replies 3 ·
Replies
3
Views
3K
  • · Replies 6 ·
Replies
6
Views
2K
Replies
4
Views
2K
Replies
15
Views
2K
  • · Replies 2 ·
Replies
2
Views
2K